crim question

¡Supera tus tareas y exámenes ahora con Quizwiz!

Marcy is a wealthy individual who makes millions of dollars each year. When it comes time to file and pay her federal income tax, Marcy refuses to disclose her income on her tax return and does not pay the federal government the required amount of money. At around the same time, Marcy looked at various websites that give false reasons for why wealthy individuals are not required to pay income taxes. Eventually, the IRS comes calling. Marcy maintains that she did not know that she was legally required to pay the income tax. Nonetheless, federal prosecutors charge her with tax evasion for failing to declare her income and pay tax on it. Can Marcy's lawyer argue mistake of law to avoid criminal responsibility? A (A) Yes, as long as she believed the information on the website (B) No, because tax evasion is a strict liability crime and does not require knowledge of the law (C) No, because mistake of law is never a defense to federal crimes (D) Yes, but only if she relied on the advice of an attorney

A Rationale: For most crimes, mistake of law is not a successful defense. The one exception is crimes where the material elements require some mental attitude with regard to the law. In those cases, the mistake of law negates the mental element. The most typical case would be a crime that requires that the defendant at least have knowledge that his or her actions violate the law—the classic example being tax evasion. The mistake of law negates the requirement that the defendant knows that the action is illegal. So, Choice A is the best answer because if Marcy did not believe the information on the website, then she would not be laboring under a mistake of law. Choice B is incorrect because tax evasion is not a strict liability crime. Choice C is incorrect because mistake of law is sometimes—though rarely—a defense to a federal crime. Finally, Choice D is incorrect because an individual could be mistaken about the law without having consulted an attorney. Recommended Reading: Pages 267-271 of Kaplan, Criminal Law: Cases and Materials, 8th Edition

Felicity opens the door of her house. Standing in front of the house is a solicitor asking for donations for a local charity. Felicity interprets the request as a demand for money that will be backed up by physical force if she does not comply. Instead of turning over the money, Felicity pulls out a gun and kills the solicitor. Felicity is charged with murder but wants to assert self-defense. Which statement is most accurate? (A) Felicity will be justified if she believed that she faced an imminent threat of serious bodily injury and her belief was reasonable. (B) Felicity will be justified if she believed that she faced an imminent threat of serious bodily injury and her belief was sincere. (C) Felicity will be justified if she believed that she faced an imminent threat of any bodily injury and her belief was reasonable. (D) Felicity will be justified if she believed that she faced an imminent threat of any bodily injury and her belief was sincere.

A Rationale: In order to be justified in using deadly force for self-defense, the actor must reasonably believe that he or she faces an imminent threat of serious bodily injury. An unreasonable belief is insufficient to meet the standard, as is a non-serious injury. Consequently, Choice A is the best answer. Choice B is incorrect because it falsely implies that a sincere belief is enough. Choice C is incorrect because it falsely suggests that any bodily injury is enough, when in fact the standard is more demanding. For the same reason, Choice D is incorrect; it is also incorrrect because it falsely suggests that a sincere belief is enough. Recommended Reading: Pages 557-561 of Kaplan, Criminal Law: Cases and Materials, 8th Edition

The police investigate a report of a kidnapping—a man who was taken from his home and held for a day before being rescued. The defendant is charged with participation in the kidnapping. The case takes place in a jurisdiction that has adopted the Model Penal Code. At trial, the defendant seeks to argue to the jury that he was laboring under a mistake of law. Specifically, the defendant asserts that he honestly believed that he was assisting law enforcement officers in making a lawful arrest of the man. Should the defendant be permitted to make this argument to the jury? A (A) Yes, because the alleged mistake negates a required mental element of kidnapping. (B) Yes, because the alleged mistake was reasonable under the circumstances. (C) No, because the mistake was unreasonable. (D) No, because it was not a mistake of fact.

A Rationale: The correct answer is Choice A. Under the Model Penal Code, a mistake of law will only be defense if it negates a required mental element of the offense, in this case kidnapping. Choice B is incorrect because whether the mistake is reasonable or not is not relevant under the MPC approach. Choices C and D are incorrect because they falsely suggest that the defense is inapplicable. Recommended Reading: Pages 258-271 of Kaplan, Criminal Law: Cases and Materials, 8th Edition

Bartholomew is charged and convicted with theft for stealing money from his employer, a restaurant that specializes in hamburgers. At his sentencing, Bartholomew's lawyer argues that he should not receive prison time because the crime—theft—was a nonviolent offense. The judge says he agrees that prison time is not appropriate. He orders Bartholomew to dress up in a hamburger costume and carry a sign that reads: "I am a thief. I am a hamburglar." The judge orders Bartholomew to do this eight hours a day on Saturdays and Sundays for an entire month, eight days in total. The judge's sentence is: C (A) Likely to be upheld on appeal if the appeals court is convinced that it is an example of integrative shaming. (B) Likely to be upheld on appeal if the appeals court is convinced that it is an example of disintegrative shaming. (C) Likely to be overturned on appeal because it is based solely on humiliation. (D) Likely to be overturned on appeal because deterrence and retribution are the only two legitimate justifications for punishment.

A Rationale: The judge's sentence is an example of a shaming penalty, which is designed to heap shame on the defendant for his illegal conduct. Choice A is the best answer because shaming penalties are often upheld when courts consider them examples of "integrative shaming," i.e. they allow the defendant to return as a productive member of society after the shaming penalty is completed, as opposed to being permanently ostracized by the community. Choice B is therefore not correct because if an appeals court viewed the penalty as an example of disintegrative shaming, the court would be more likely to overturn the sentence rather than confirm it. Choice C is also incorrect because the existence of humiliation, by itself, is not a reason to overrule a punishment and courts have frequently upheld punishments that involve some level of humiliation of the defendant, as long as the shaming is not disintegrative. Choice D is also incorrect because, although deterrence and retribution are the major justifications for punishment, they are not the only ones. Recommended Reading: Pages 75-76 of Kaplan, Criminal Law: Cases and Materials, 8th Edition

Josephine is charged with assaulting a rival drug dealer, named Ronald, on a street corner. Josephine's assault on Ronald rendered him unconscious and he fell down on the street corner, where he lay bleeding for 10 minutes. At that point, a second man named George is walking down the street. George encounters the unconscious Ronald and looks at him. After contemplating the situation and realizing that Ronald will offer little resistance, George robs and kills Ronald. Is Josephine a but-for cause of Ronald's death? (A) Yes, if it is the case that "but for" Josephine's assault, Ronald would never have been on the sidewalk and would not have been attacked by George. (B) Yes, but only if the standard for proximate cause is also satisfied. (C) No, because two people can never be responsible for one murder. (D) No, because Josephine never intended for Ronald to die.

A Rationale: The materials on causation are complex and should be approached carefully. It is crucial to separate out the standards for but-for causation and proximate causation, which are two entirely separate strands of analysis. But-for causation involves a simple test of whether the result would have happened if the defendant had not acted. It is completely separate from proximate cause, which asks whether the defendant's actions were close enough to the final result to make the defendant responsible for the resulting harm. So, Choice A is the correct answer because if Josephine had not first attacked Ronald, he never would have been on the sidewalk for the second person to kill him. Choice B is incorrect because proximate cause is a separate issue that isn't implicated by this question. Choice C is incorrect because the existence of a second individual responsible for the murder might affect proximate cause but not but-for causation. Choice D is incorrect because causation does not depend on whether Josephine acted with the mental state of purpose. Recommended Reading: Pages 292-299 of Kaplan, Criminal Law: Cases and Materials, 8th Edition

Victoria is trapped in an abusive relationship with her husband, John, who has subjected her to a daily torrent of verbal, emotional, and physical abuse over the last 10 years. On two occasions, Victoria has tried to leave John but was unsuccessful. The first time, Victoria went to stay with her sister, but John came and dragged her from the sister's home and carried her back to their marital home. The second time, Victoria went to a motel, but the same thing happened. Victoria called the police 10 times over the course of several years after being hit by John, but she was too scared to testify against him so the charges were always dismissed. One day, John got very drunk and threatened to kill Victoria. Then John fell asleep. Worried that John would kill her when he woke up, Victoria took his gun and killed him while he was sleeping in his bed. Victoria is charged with murder. Which of the following statements is correct about how her trial might unfold? (A) All courts will allow introduction of evidence regarding Battered Person's Syndrome to demonstrate self-defense. (B) Evidence of Battered Person's Syndrome might be introduced at trial as an argument for mitigating punishment. (C) Most courts recognize an independent justification called Battered Person's Defense. (D) Both A and B.

B Rationale: Most courts do not recognize a new defense called Battered Person's Defense, although evidence regarding the syndrome is sometimes admissible as relevant in these cases. Consequently, Choice C is not the correct answer. Choice A is incorrect because not all courts will allow introduction of the testimony to help establish that the defendant acted under a reasonable belief. Choice B is the best answer because evidence of the syndrome will at least be relevant during the sentencing phase as a possible mitigating factor that might determine the appropriate length of imprisonment or other punishment. Recommended Reading:

Donald receives a phone call from an anonymous caller who refuses to identify himself. Donald is directed by the caller to sexually assault his neighbor. The caller tells Donald that if Donald fails to comply with the demand, the caller will track down and find Donald's best friend and "punch him." Fearful of his best friend being punched, Donald succumbs to the threat and assaults the neighbor. Donald is charged with assault but wishes to assert the duress defense. Which statement is most correct? (A) The duress defense can never apply in cases of sexual assault. (B) Donald will not be excused by duress if the jury concludes that a person of reasonable firmness in this situation would have been able to resist the threat. (C) Donald is not excused by duress because the sexual assault was not the lesser of two evils. (D) None of the above.

B Rationale: Only some threats will generate a duress defense. If a person of reasonable firmness would be capable of resisting the threat, then the excuse does not apply. In this case, it seems plausible that a jury might conclude that a person of reasonable firmness would be able to resist the threat of a friend simply being punched (as opposed to being injured more severely). Consequently, Choice B is the best answer. Choice A is incorrect because there is no bright line rule regarding duress and sexual assault. Choice C is also not the best answer because although the sexual assault was not the lesser of two evils, duress generally does not require a balancing of evils. Choice D is also incorrect because it precludes Choice B, which was the correct answer.

Oskar is at work and gets into a dispute with his co-worker. The co-worker screams at him, "You did such a bad job on that assignment -- I wish you would just go away and die!" Oskar responds by pulling out a gun and shooting the co-worker, who dies. Oskar is charged with murder. At trial, Oskar argues that he was justified by self-defense because he believed that he faced an imminent threat of death from his co-worker. If the jury concludes that Oskar's belief was sincere but unreasonable, of what crime will Oskar be convicted in a jurisdiction that has not adopted the Model Penal Code? (A) Murder, if the jurisdiction recognizes the doctrine of imperfect self-defense (B) Manslaughter, if the jurisdiction recognizes the doctrine of imperfect self-defense (C) Manslaughter, if the jurisdiction does not recognize the doctrine of imperfect self-defense (D) Negligent homicide

B Rationale: Some jurisdictions recognize the doctrine of imperfect self-defense but others do not. It is helpful to think of a switch with either two or three positions. In some jurisdictions there are only two options: Either the defendant is guilty of murder or not guilty because they are justified by self-defense. In order for the defendant to be not guilty, they must meet all of the standards for self-defense, including the reasonable belief standard. So because Oskar's belief is unreasonable, in that jurisdiction he would be guilty of murder. However, some jurisdictions have a middle option for just these cases. Under the doctrine of imperfect self-defense, those individuals acting pursuant to an unreasonable belief are downgraded from murder to manslaughter. Consequently, the best answer is Choice B. Choice A is incorrect because if the jurisdiction recognizes imperfect self-defense, Oskar will be convicted of manslaughter, not murder. Choice C is incorrect because if the jurisdiction does not recognize the doctrine of imperfect self-defense, he would be guilty of murder, not manslaughter. Choice D presents an option (negligent homicide) that is not available unless the jurisdiction has adopted the Model Penal Code (Sec. 3.09). Recommended Reading: Page 560 of Kaplan, Criminal Law: Cases and Materials, 8th Edition

Gina is the operator of a carnival park that contains several large amusement park rides such as rollercoasters and other attractions. One of the rollercoasters is called the Tornado and includes several fast twists and turns. One day, a rider named Jeremy is thrown from the rollercoaster and dies. A police investigation reveals that the seatbelt broke. Further investigation revealed that 12 months prior to the accident, Gina considered upgrading the seatbelts of the cars but declined to do so because she did not want to pay the cost of the upgrade. How might the Model Penal Code assess whether Gina's actions were a proximate cause of the rider's death? (A) It states that operators of dangerous machines are strictly liable for the resulting injuries. (B) It asks whether the resulting death was not too remote or accidental to have a just bearing on Gina's criminal liability. (C) It asks if it was reasonably foreseeable that a rider might die from a broken seatbelt. (D) It asks whether the result was "closely connected" to the defendant's conduct.

B Rationale: The Model Penal Code, in section 2.03(3), states that in recklessness and negligence cases, "the element is not established if the actual result is not within the risk of which the actor is aware or, in the case of negligence, of which he should be aware unless: (a) the actual result differs from the probable result only in respect that a different person or different property is injured or affected or that the probable injury or harm would have been more serious or more extensive than that caused; or (b) the actual result involves the same kind of injury or harm as the probable result and is not too remote or accidental in its occurrence to have a [just] bearing on the actor's liability or on the gravity of his offense." Consequently, Choice B is the best answer. Choice A is incorrect because the MPC has no such rule regarding dangerous machines. Choice C has a whiff of plausibility because non-MPC jurisdictions frequently understand proximate cause in terms of "reasonable foreseeability," but it is not the MPC test for causation. Similarly, Choice D sounds plausible but is not the MPC test. Recommended Reading: Pages 307-309 of Kaplan, Criminal Law: Cases and Materials, 8th Edition

Remus suffers from a mental illness and he is a resident of a voluntary in-patient psychiatric facility. After living at the facility for 12 months, Remus is discharged with a prescription for an anti-psychotic medication. Remus does well for several months until he stops taking his medicine and his old demons return. One day, Remus breaks into a house by smashing the front window. The police arrest him and the local police charge him with burglary. However, Remus and his lawyer argue that he cannot be convicted of burglary because Remus was delusional and falsely believed that someone inside the house was being attacked by aliens and Remus needed to save the homeowner from imminent extraterrestrial peril. Remus's lawyer wants him to plead not guilty by reason of insanity, but Remus is offended by this suggestion because he doesn't consider himself insane. He wants to plead not guilty; however, he still wants his psychiatrist to testify that he was incapable of forming the required mens rea for the offense that he was charged with. Can this evidence be admitted? B (A) Yes, if the jurisdiction recognizes the M'Naghten test (B) Yes, if the jurisdiction allows the doctrine of diminished capacity (C) No, because in every jurisdiction psychiatric testimony is limited to cases where the defendant pleads not guilty by reason of insanity (D) Yes, if the jurisdiction applies the irresistible impulse test

B Rationale: The answer to this question depends on the jurisdiction. Essentially, Remus wants to introduce psychiatric evidence without pleading "not guilty by reason of insanity." Several jurisdictions prohibit the introduction of such evidence. However, some jurisdictions allow it under the doctrine of diminished capacity so that the evidence can be used to demonstrate that the defendant did not act with the required mental element for the offense. In this case, the testimony could establish that Remus's mental condition prevented him from having the mens rea to commit a felony inside the house, an essential ingredient of a burglary case. Consequently, Choice B is the best answer. Choice A and Choice D are both incorrect because the irresistible impulse and M'Naghten tests are not relevant. Choice C is not the best answer because the exclusion of this evidence only exists in some jurisdictions, not all jurisdictions. Recommended Reading: Pages 227-281 of Kaplan, Criminal Law: Cases and Materials, 8th Edition

Gary shoots at a police officer. The police officer dies. At trial, Gary's lawyer must decide between asserting self-defense or insanity as defenses to the charge of murder. Which categories most accurately describe these defenses? (A) Insanity is a justification while self-defense is an excuse. (B) Self-defense is a justification while insanity is an excuse. (C) Self-defense and insanity are both justifications. (D) Self-defense and insanity are both excuses.

B Rationale: The classic definition of a justification is a defense that negates the wrongfulness of the act. Consequently, self-defense is considered a paradigmatic justification because an actor who uses force in self-defense is not acting wrongly. In contrast, the classic definition of an excuse is a defense that negates the culpability of the actor. Consequently, insanity is a classic example of an excuse. So Choice B is the best answer. Choice A is incorrect because insanity is not a justification. For similar reasons, Choices C and D cannot be correct. Recommended Reading: Pages 549-551, 557-560, and 647-649 of Kaplan, Criminal Law: Cases and Materials, 8th Edition

A woman gets into a dispute with a man on the street. The woman hits the man on the head, ultimately resulting in the man's death. The woman is charged with intentional murder and manslaughter by the prosecutor. The woman asserts a claim of self-defense, arguing that she believed that the man was about to attack her. The fact-finder concludes that the woman was genuine but mistaken in her belief and that she was reckless in the formation of her erroneous belief that the man was a threat to her. In a jurisdiction that applies the Model Penal Code, what result if the woman argues imperfect self-defense to the trial court? (A) Conviction for intentional murder (B) Conviction for manslaughter (C) Acquittal on both charges (D) The MPC does not cover this scenario

B Rationale: The correct answer is Choice B. Section 3.09(2) of the Model Penal Code allows for imperfect self-defense, but if the defendant is reckless in the formation of the belief, the defense cannot be used for any offense for which recklessness would suffice to establish culpability. Choice A is incorrect because the defendant can use the defense for a charge of intentional murder. Choice C is incorrect because the defendant would not be exonerated for the charge of reckless manslaughter. Choice D is incorrect because the MPC has a provision covering this exact scenario. Recommended Reading: Pages 578-579 of Kaplan, Criminal Law: Cases and Materials, 8th Edition

Late one Sunday night, Marino is watching TV in the basement of the home he rents. He hears glass breaking and runs up the stairs. He sees that an intruder has broken into the front hallway and is standing there empty-handed. Marino pulls out a gun and shoots the intruder. The prosecutor charges Marino with murder. However, Marino's lawyer asserts self-defense and defense of habitation. Which of the following statements is most correct? (A) Marino is guilty because he had a duty to retreat and could have retreated safely. (B) Marino is not guilty because he had no duty to retreat in his castle. (C) Marino had a duty to retreat because the intruder was apparently unarmed. (D) Marino had a duty to retreat because he was renting the house.

B Rationale: The general rule in self-defense is that deadly force is only permissible if it is necessary to defend against a threat of imminent death or great bodily harm. If the defender can safely retreat, then deadly force is not truly necessary. But some jurisdictions never require retreat, and in those that do there is an exception for cases where the defender is in his or her own home (his ''castle''). Likewise under the Defense of Habitation doctrine recognized in some jurisdictions deadly force can be used to prevent a forcible felony in the home, and there is no duty to retreat. Consequently, Choice B is the best answer. Choice A is incorrect for many jurisdictions and situations where there is no duty to retreat. Choice C is not a good answer because the duty to retreat in the home does not depend solely on whether the intruder was armed. Choice D is not a good answer because the Castle Doctrine and Defense of Habitation doctrines apply to renters as well homeowners. Recommended Reading: Pages 582-588 of Kaplan, Criminal Law: Cases and Materials, 8th Edition

Frank is convicted of two counts of first-degree premeditated murder. During the sentencing phase, the prosecutor makes the following statement: "Frank deserves to be punished for a very long time. His actions—killing two innocent people—represent the worst possible moral violation possible. We will never be able to bring the victims back. They are gone forever. Frank deserves to suffer for his crimes and suffer for a long time." The prosecutor's argument is: D (A) Based on the notion of deterrence. (B) Retributive in nature. (C) Unconstitutionally vague, because it appeals to morality. (D) Both B and C.

B Rationale: The prosecutor's argument about Frank's sentence appeals mostly to retributivism because the prosecutor talks about the suffering that Frank deserves. Consequently, Choice A is incorrect because the prosecutor does not reference deterrence, or the idea that punishment is justified because it will deter the offender, or others, from committing future crimes. Choice B is therefore the correct answer. Choice C is incorrect because there is no constitutional prohibition preventing the prosecutor or the judge from referencing morality during a criminal sentencing hearing. Choice D is therefore also incorrect because Choice C is incorrect. Recommended Reading: Pages 31-34 of Kaplan, Criminal Law: Cases and Materials, 8th Edition

Adrienne robs a bank. After leaving the bank with the money, she hops into a car and drives away at a high rate of speed. Because the bank teller activated a silent alarm, the police arrive quickly and several police cars join in a high-speed chase of Adrienne's car, which is now driving down a busy street. One of the police officers is driving negligently and carelessly. Eventually, the police office loses control of his car, which goes into a ditch and then is launched into the air. The out-of-control police car hits an innocent pedestrian who just happens to be standing nearby. The prosecutor wants to charge Adrienne with manslaughter in connection with the death of the pedestrian. Does the action of the police officer break the chain of causation leading back to Adrienne? (A) Yes, because the police officer committed the last physical act in the chain of causation (B) No, because it is foreseeable and not highly abnormal for a police officer to act negligently during a car chase (C) Yes, because intervening agents who act negligently always break the chain of causation (D) No, because Adrienne committed an aggravated felony by robbing the bank

B Rationale: This is a problem of proximate cause. Although the police officer is an intervening agent, the question is whether the intervening agency makes it impossible to hold the original actor, Adrienne, responsible for the ultimate result (the death of the pedestrian). The general rule is that intervening agents who act negligently do not break the chain of causation if the negligence is foreseeable and not highly abnormal. Choice B is therefore the correct answer. Choice A is incorrect because the fact that the police officer committed the last physical act does not determine whether the chain of causation back to Adrienne is preserved or negated. Choice C is incorrect because it is simply false to say that intervening agents who act negligently always break the chain of causation; in many situations, negligent actors do not break the chain of causation. Finally, Choice D is incorrect because the classification of the original crime as an aggravated felony—or some other classification—has no bearing on the legal question of proximate cause. Recommended Reading: Pages 311-326 of Kaplan, Criminal Law: Cases and Materials, 8th Edition

Baxter works at an investment bank. As part of his job, he helps clients with their financial transactions. Since some of the clients are very wealthy, the transactions are sometimes very complex. On one occasion, a client asks him for help transferring money from one account to another account. Baxter executes the transfer. The next day, the police arrest the client and charge him with money laundering. Apparently, the money came from drug smuggling, and prosecutors believe that the client is involved with organized crime. The prosecutors also charge Baxter with money laundering since he was the one who transferred the money between the accounts. The state's money laundering statute includes the following provision: "Whoever engages in a financial transaction with knowledge that the transaction will conceal the existence of illegal activity is guilty of the crime of money laundering." The state also has a mistake of fact provision borrowed from the Model Penal Code. Baxter claims that he was unaware of the underlying illegal activity. Can he argue mistake of fact to avoid criminal liability? B (A) No, because ignorance of the law is no excuse (B) Yes, because his ignorance negated the required mental state for the crime (C) Yes, because he did not act with the purpose to conceal the illegal activity (D) No, because the offense in question is a strict liability crime

B Rationale: This jurisdiction follows the Model Penal Code scheme for mistake of fact. So, in order to determine whether a mistake of fact is relevant, one must first examine the material elements of the statute for the crime in question. In this case, the statute requires that the defendant have knowledge "that the transaction will conceal the existence of illegal activity." The required mental state is acting knowingly. Choice A is incorrect because "ignorance of the law is no excuse" is a reference to the defense of mistake of law, not the defense of mistake of fact. Choice C is not the best answer. Although Baxter did not act with purpose, as Choice C suggests, this isn't dispositive because the statute only requires acting with knowledge, not acting with purpose. Choice D is also incorrect because the statute is not a strict liability crime; in fact, the statute requires a substantial mental state (knowledge). Therefore, Choice B is the correct answer. Baxter could argue that he was mistaken about the consequences of the transaction and that his mistake negated the required mental state of acting with knowledge. Recommended Reading: MPC Sec. 2.04

Marianna's arm hits Jackson in the jaw causing significant damage and requiring medical attention. The police arrive and Marianna concedes that it was her arm that caused the damage to Jackson's jaw. The police charge Marianna with assault. At trial, Marianna offers uncontested evidence that the physical movement of her arm was the result of an epileptic seizure. Did the state's proof satisfy the actus reus requirement? B (A) Yes, because an epileptic seizure is an act (B) No, because the crime presupposes a voluntary act and an epileptic seizure is involuntary (C) Yes, because Marianna was not born with epilepsy but developed it later in life (D) Yes, because assault is a strict liability crime

B Rationale: This question focuses on the act requirement. Although Marianna hit the victim, her act was involuntary and did not constitute a voluntary act. Consequently, Choice A is not correct. What matters is whether a seizure is a voluntary act. Choice B is therefore the correct answer. Choice C is incorrect because when Marianna developed the illness does not matter and does not change whether it should be classified as voluntary or involuntary. Also, Choice D is incorrect because assault is not a strict liability crime. Recommended Reading: Pages 153-159 of Kaplan, Criminal Law: Cases and Materials, 8th Edition

Judge Rana is sentencing a defendant who committed armed robbery and stole more than $10,000 from a bank. The defense lawyer in the case tells the judge that his client should receive no prison time. To support his argument, he notes that his client was severely injured during the bank robbery and is now physically incapable of repeating the crime, so a prison term is not needed to convince the defendant not to return to a life of crime. Is this a retributive argument? B (A) Yes, because retributive arguments always entail that defendants receive no prison time (B) No, because the lawyer's argument is about deterrence, not retribution (C) No, because the judge is unlikely to agree with the argument (D) Yes, because criminal punishment is never morally justified

B Rationale: This question focuses on the difference between retributivism and deterrence. Retributive theories justify the institution of punishment based on the idea that the guilty deserve to suffer for their wrongdoing. In contrast, deterrence arguments are more utilitarian in nature: they are based on the idea that the prospect of future punishment will deter someone from committing a crime. So Choice A is wrong because retributivism might entail a harsh punishment or a light punishment, depending on the circumstances. Choice B is correct: the defense lawyer is making an argument about deterrence because he is saying that a prison term isn't necessary to deter his client from committing a future crime. Choice C is incorrect because whether the judge agrees or not with the argument has nothing to do with whether it is based on retributivism or utilitarianism. Choice D is incorrect because the question was about what type of argument the lawyer was making, not whether criminal punishment is morally justified or not. Recommended Reading: Pages 31-32 of Kaplan, Criminal Law: Cases and Materials, 8th Edition

Alice, Bill, and Cindy are all in the same room together. Alice has a gun. Billy is standing right in front of Cindy. Alice fires the gun. The bullet hits Billy, travels through his body, and then hits Cindy. Alice wanted the bullet to hit Billy. Alice was aware that the bullet was virtually certain to hit Cindy. Which statement best describes Alice's mental state? B (A) She killed Billy with knowledge and Cindy with purpose. (B) She killed Billy with purpose and Cindy with knowledge. (C) She killed Billy with knowledge and Cindy with recklessness. (D) She killed Billy with purpose and Cindy with negligence.

B Rationale: This question turns on the distinction between acting with purpose and acting with knowledge. Since Cindy desired the death of Billy, that killing was performed with purpose. However, her mental state with regard to Cindy is only knowledge because she was aware that the bullet was virtually certain to hit Cindy. Choice A is the wrong answer because the names are reversed. Choice B is the best answer because the killing of Billy was performed with purpose. Choice C is incorrect because it understates Alice's mental state with regard to both Billy and Cindy. The killing of Cindy was performed with knowledge, not recklessness, because Alice was aware that the harm was "practically certain" to occur. Choice D is incorrect because the killing of Cindy was more than negligent—it was performed with knowledge. Recommended Reading: Pages 231-235 of Kaplan, Criminal Law: Cases and Materials, 8th Edition

Vanessa is a troubled individual who suffers from delusions and schizophrenia. Vanessa hears voices in her head. On many occasions, the voices tell her that the government is secretly recording her thoughts and that her friends and family members are plotting against her. In order to eliminate these thoughts, Vanessa had been taking powerful anti-psychotic medications, but she stopped taking the medications several months ago because she found the side effects burdensome. Vanessa decides that the only way to stop the government is to blow up the federal office building located in her hometown. On her way to commit this crime, Vanessa walks by a jewelry store. Vanessa is immediately taken with a beautiful men's diamond ring in the window. Vanessa walks into the store and asks how much the ring is. The storekeeper says it costs $5,000. Vanessa desperately wants to buy the ring for her husband, but she does not have enough money. Instead, Vanessa smashes the glass case and takes the ring. The police arrive a few minutes later and tackle Vanessa on the street. She is charged with theft. Which of the following statements is most correct? (A) Vanessa will be excused, because she was hearing voices. (B) Vanessa will be excused, because a psychiatrist will be able to testify that she suffers from specific psychiatric disorders. (C) Vanessa will not be excused, because her insanity played no role in her decision to steal the ring. (D) Vanessa will not be excused if the jurisdiction applies the MPC "substantial capacity" standard. Rationale: Although Vanessa is clearly mentally ill, her psychiatric condition did not prevent her from appreciating the wrongfulness of her action (stealing the ring) or from understanding the nature of her action. Also, the psychiatric condition did not prevent her from conforming her behavior to the requirements of the law. Of course, the psychiatric condition did play a role in her decision to blow up the federal building, but she never got there and wasn't charged with that crime. The theft of the ring had nothing to do with the voices in her head. Consequently, Choice C is the best answer. Choice A is wrong because just hearing voices is not enough to make a successful insanity claim. Choice B is wrong because psychiatric testimony is a necessary but not sufficient condition for succeeding with the insanity defense. Choice D is wrong because it does not matter whether the jurisdiction applies the MPC provision or not because the real issue is the lack of connection between the insanity and the theft.

C . Rationale: Although Vanessa is clearly mentally ill, her psychiatric condition did not prevent her from appreciating the wrongfulness of her action (stealing the ring) or from understanding the nature of her action. Also, the psychiatric condition did not prevent her from conforming her behavior to the requirements of the law. Of course, the psychiatric condition did play a role in her decision to blow up the federal building, but she never got there and wasn't charged with that crime. The theft of the ring had nothing to do with the voices in her head. Consequently, Choice C is the best answer. Choice A is wrong because just hearing voices is not enough to make a successful insanity claim. Choice B is wrong because psychiatric testimony is a necessary but not sufficient condition for succeeding with the insanity defense. Choice D is wrong because it does not matter whether the jurisdiction applies the MPC provision or not because the real issue is the lack of connection between the insanity and the theft.

Fabian is a police officer assigned to regular patrol with a police vehicle. One day, Fabian receives a 911 alert from his dispatcher about a report of an intoxicated individual wandering in the street and making violent threats. Fabian arrives on the scene, walks toward the man described in the 911 call, and sees that the man is holding a black object in his hand. Fabian believes the black item is a gun and responds by quickly shooting the man. After the man falls to the ground, Fabian realizes that the black item is a cell phone, not a gun. The local prosecutor believes that Fabian does not meet the standards for self-defense in this jurisdiction because his belief was objectively unreasonable. Fabian's lawyer argues that this is not the end of the analysis. Which of the following statements is most correct? (A) Police officers are always subject to the same standards as civilians for the use of deadly force. (B) Police officers are always justified in using force as long as they believe that they are acting lawfully. (C) Many jurisdictions have statutory defenses for police officers that are broader in scope than regular self-defense. (D) A police officer is never justified in killing an unarmed suspect.

C Rationale: Although police officers can assert the regular defense of self-defense if their conduct meets the statutory and case law requirements, police officers are also protected by additional statutory defenses in many jurisdictions. These vary widely from jurisdiction to jurisdiction. In the case described above, some jurisdictions would limit the analysis to whether Fabian's actions met the standard for self-defense, although even these jurisdictions would normally not require the officer to retreat even if that would be required of a non-police officer. Other jurisdictions would apply a specific statutory defense for the lawful exercise of deadly force by a police officer. Consequently, the best answer is Choice C because it correctly refers to these statutes. Choice A is far too categorical and fails to recognize the existence of specific defenses for police officers. Choice B and Choice D are both incorrect because they are too categorical. Choice B assumes that police officers are always justified while Choice D assumes that police officers are never justified in situations where the suspect is unarmed. The truth of the matter is that such killings are sometimes justified and sometimes not. Recommended Reading: Pages 594-599 of Kaplan, Criminal Law: Cases and Materials, 8th Edition, and Model Penal Code Secs. 3.04 and 3.07

Bob is at his friend's house and is very intoxicated after smoking a large amount of marijuana. The police arrive at the house and drag him from inside the doorway of his house into the street. Bob is arrested there and charged with appearing in public while in an intoxicated state, a misdemeanor in that state punishable by up to one year in jail. Can Bob be convicted of this offense? C (A) Yes (B) Yes, as long as his sentence is only a fine and does not include a jail term (C) No, because Bob's "appearance in public" was not a voluntary act (D) No, because the police conduct violated his constitutional right against self-incrimination

C Rationale: Although the statute says nothing explicitly about a voluntary appearance, criminal statutes are interpreted to require a voluntary act. The police carrying Bob from inside the house to the street transformed what would otherwise be a voluntary act into an involuntary act. Therefore, Bob never committed an act and should not be convicted of a criminal offense. Consequently, Choice A is wrong. Choice B is wrong because the outcome has nothing to do with the severity of the punishment. Choice D is also wrong because the right against self-incrimination is not implicated by the police conduct in this case. Choice C is correct. Recommended Reading: Pages 152-153 of Kaplan, Criminal Law: Cases and Materials, 8th Edition

Felicia is the owner of a rare painting by Vincent Van Gogh. It is hanging in the hallway of her apartment in a building on Park Avenue in Manhattan. The painting is estimated to be worth approximately $25 million. One day, Felicia realizes that the painting needs historic preservation to repair some cracks in the frame of the painting. She arranges for a company to pick up the painting and transport it to the Metropolitan Museum of Art, which will repair the cracks in exchange for the right to display the painting in its exhibition for the next 10 years. Unfortunately, someone at the transportation company has leaked the information about the painting, and two thieves decide to steal the painting. As the painting is being loaded onto the truck on Park Avenue, the two thieves steal the painting from the man loading it into the truck. As the thieves are quickly walking away with the painting, Felicia comes down to the street and starts running after the thieves. At this point, the thieves are continuing to run away with the painting, and Felicia realizes that the only way to stop them from absconding with the painting is to pull out her gun and shoot both of them in the back. Which statement is most correct? (A) Felicia had no right to use deadly force solely to defend her property. (B) If the thieves were armed and Felicia was aware of that fact, deadly force might have been justified to defend herself but that justification evaporated when the thieves ran away and the threat of physical violence dissipated. (C) Both A and B. (D) Felicia had no right to use deadly force because the property was officially on loan to the Metropolitan Museum of Art.

C Rationale: Generally, a defender is not permitted to use deadly force to protect property, even valuable property. The fact that the painting is worth $25 million is immaterial. Consequently, Choice A is plausible. Choice B is also plausible because if the thieves were armed, they might have been subject to deadly force if someone believed that they faced a threat of death or serious bodily injury from the thieves, but that entitlement to use deadly force would have evaporated once the thieves ran away. So Choice C is the best answer because both A and B are correct statements. Choice D is incorrect because the fact that Felicia loaned the painting to the museum is totally immaterial. Deadly force is not permitted for the mere protection of property. Recommended Reading: Pages 599-605 of Kaplan, Criminal Law: Cases and Materials, 8th Edition

Billy is walking through a park when he is confronted by someone who tries to take his wallet by force. Billy struggles with the robber and then takes out his gun and shoots and kills the robber. Billy is charged with murder but claims self-defense. Specifically, his lawyer claims that Billy's force was appropriate because the jurisdiction recently passed a Stand Your Ground law. Which statement is most accurate? (A) Billy had a duty to retreat because he was outside of his castle. (B) Billy had no duty to retreat because a park is the same as a castle. (C) Billy had no duty to retreat because he reasonably believed that force was necessary to prevent the commission of a forcible felony. (D) Both A and B.

C Rationale: In the absence of a Stand Your Ground law, the duty to retreat applies unless the actor is inside his dwelling (the Castle Doctrine or Defense of Habitation). At first glance, therefore, Choice A seems like a good answer. However, this jurisdiction applies the Stand Your Ground law, which broadens the Castle Doctrine exception to the duty to retreat. Most of these statutes broaden the exception in cases where the actor is facing a forcible felony. Consequently, Choice C is the best answer. Choice B is not correct because there is no provision that would turn a park into a castle. Choice D is incorrect and internally contradictory. Recommended Reading: Pages 585-586 of Kaplan, Criminal Law: Cases and Materials, 8th Edition

Which statement best describes the distinction between justifications and excuses? (A) Justifications negate the wrongfulness of the act while excuses negate the culpability of the actor. (B) Justifications can apply to all actors in a situation, while excuses are personal to particular actors. (C) Both A and B. (D) Neither A nor B.

C Rationale: The classic distinction between justifications and excuses is that the former negates the wrongfulness of the act while the latter negates the culpability of the actor. So, for example, self-defense applies not just to the actor using defensive force but all those who might assist him or her. If one of them is justified, so are all of them. In contrast, insanity would apply just to the actor and would not automatically excuse the conduct of other sane participants in the criminal endeavor. Consequently, Choice A and Choice B are both correct, and the best answer is Choice C. For the same reason, Choice D cannot be correct. Recommended Reading: Pages 549-551 of Kaplan, Criminal Law: Cases and Materials, 8th Edition

A police officer is chasing a suspect that the police officer reasonably believes was responsible for the brutal murder of another police officer. The police officer shoots the suspect in the back to prevent him from escaping. Although the suspect was not directly threatening the police officer at the time of the shooting, the police officer had no other alternative to stop the fleeing suspect. Before shooting, the police officer announced himself as a police officer and warned the suspect that he would shoot him unless he stopped his flight. Was the killing unconstitutional under the Fourth Amendment? (A) Yes, because the police officer shot him in the back. (B) Yes, because the suspect was not threatening the police officer. (C) No, because the police officer was entitled to consider the suspect dangerous because he had probable cause to believe that he had committed a violent crime. (D) No, because the Fourth Amendment is irrelevant to cases involving the use of force by the police.

C Rationale: The correct answer is Choice C. According to the framework announced in Tennessee v. Garner, 471 U.S. 1 (1985), the police may only use deadly force if the suspect poses a danger to the police officer or the public. However, the police officer is entitled to infer that dangerousness if he has probable cause to believe that the suspect has engaged in a crime involving the infliction of serious physical harm. The murder of the previous police officer would satisfy this standard. Choices A and B are incorrect because they falsely label the shooting as a violation of the Fourth Amendment. Choice D is incorrect because it falsely states that the Fourth Amendment plays no role in law enforcement use-of-force cases. Recommended Reading: Pages 594-601 of Kaplan, Criminal Law: Cases and Materials, 8th Edition

The Model Penal Code's test for insanity indicates that, "A person is not responsible for criminal conduct if at the time of such conduct as a result of mental disease or defect he lacks substantial capacity either to appreciate the criminality [wrongfulness] of his conduct or to conform his conduct to the requirements of the law." What is the meaning of the brackets around the word "wrongfulness"? (A) The drafters of the MPC wanted to give judges in each trial the discretion to select either "criminality" or "wrongfulness" as the standard. (B) The drafters of the MPC wanted to give defendants in each trial the discretion to select either "criminality" or "wrongfulness" as the standard. (C) The drafters of the MPC wanted to give the legislature in each state the discretion to select either "criminality" or "wrongfulness" as the standard. (D) None of the above.

C Rationale: The drafters of the Model Penal Code could not decide whether the standard should state "criminality" or "wrongfulness," so they decide to complete the provision with A and B options for each jurisdiction to select between. Consequently, some jurisdictions go with criminality while other jurisdictions go with wrongfulness. Choice C is therefore the best answer. Choice A is incorrect because it would chaotic to apply different standards in different trials. Choice B is incorrect because it would be weird to allow defendants to select the standard that would exonerate them. Choice D is wrong because it precludes C, which is the correct answer. Recommended Reading:

Valerie has been undergoing medical treatment for several years for a mild mental illness. She takes medication, but the medication is only partially successful in repressing her symptoms. One day, Valerie is shopping at Walmart and takes a large flat-screen TV from the shelf and places it in her shopping cart. She rolls the shopping cart to her car and drives away. The police view security camera footage from the parking lot and see her license plate number. The police go to Valerie's house to arrest her for grand theft. At the trial, Valerie's psychiatrist testifies that her mental illness impaired her ability to distinguish between right and wrong and that's why she took the TV. On cross examination, the prosecutor asks the psychiatrist whether the mental illness totally eviscerated her ability to distinguish between right and wrong with regard to the taking of the TV. The psychiatrist concedes that Valerie's mental illness did not completely destroy her ability to distinguish between right and wrong, but that because of the illness, Valerie lacked substantial capacity to appreciate the criminality of the action. Which of the following statements is most correct? (A) Valerie is not guilty in a jurisdiction that applies the M'Naghten test. (B) Valerie is not guilty in a jurisdiction that applies the irresistible impulse test. (C) Valerie is not guilty in a jurisdiction that applies the MPC test. (D) Valerie is guilty in every jurisdiction regardless of its test for insanity.

C Rationale: The key element of the psychiatrist's testimony is that Valerie's capacity to understand the criminality of the situation was compromised by the mental illness but not completely destroyed by it. This suggests that both the M'Naghten and irresistible impulse tests would not exonerate Valerie. Consequently, Choice A and Choice B are wrong. However, Choice C is correct because the Model Penal Code applies the substantial capacity test which only requires that the actor lack the "substantial capacity" either to appreciate the criminality of her conduct or to conform her conduct to the requirements of the law. Choice D is incorrect because it is too categorical and falsely suggests that the MPC substantial capacity test is not relevant in this case. Recommended Reading: Pages 669-674 of Kaplan, Criminal Law: C

Victor hits George on the top of the head with a frying pan with the purpose of killing George. George falls to the ground and is completely unconscious. Victor sees George motionless on the ground and believes that George is dead. In fact, George is still alive. Later that day, with the intention of disposing George's body, Victor wraps up George in a rug, goes to a nearby river, and dumps it in the water. At this point, George drowns and dies. Can Victor be prosecuted for intentional murder? (A) Yes, because he acted with the purpose to kill George (B) Yes, because he acted with knowledge (C) No, because there was no concurrence of the elements (D) No, because he was only reckless

C Rationale: The key to answering this question is understanding that there must be a concurrence of the elements. When Victor hits George, he satisfies the act requirement for murder but the prohibited result does not occur. When Victor dumps what he thinks is a dead body in the river, he causes the prohibited result but does not satisfy the mens rea requirement of purpose or knowledge. The material elements of an offense must be concurrent, rather than patched together. (That being said, Victor is clearly guilty of both attempted murder and either involuntary manslaughter or reckless murder). Consequently, Choices A and B are incorrect; it is irrelevant that Victor acted with purpose or knowledge, given the lack of concurrence. So Choice C is the correct answer. Likewise, Choice D is incorrect because the question asks whether Victor can be prosecuted for intentional murder, not whether he is guilty of a crime of recklessness. Recommended Reading: Pages 202-210 of Kaplan, Criminal Law: Cases and Materials, 8th Edition

Jackson is charged with robbery and arson for burning down a local community center. Jackson's lawyer wants to argue that Jackson was insane during the crime and cannot be convicted of the offense. Specifically, the lawyer thinks his client needs substantial medical attention and his interests would be best served by being found not guilty by reason of insanity. Which statement is most correct about the burden of proof in insanity cases? (A) The defendant always has the burden of proving insanity by a preponderance of the evidence. (B) The prosecution is under a constitutional requirement to prove sanity beyond a reasonable doubt. (C) Jurisdictions are split on allocating the burden to the defense or the prosecution. (D) All US jurisdictions place the burden on the defendant to prove the defense of insanity by clear and convincing evidence.

C Rationale: The short answer to this question is that jurisdictions are split. Some jurisdictions require the state to prove sanity beyond a reasonable doubt, but other states require the defense to prove insanity by either a preponderance of the evidence or by clear and convincing evidence. Either way, the allocation of the burden in insanity cases shifts depending on which jurisdiction one is in. Consequently, Choice C is the only correct answer. Choices A, B, and D are all incorrect because they each suggest a uniformity of practice across jurisdictions that simply does not exist.

Heather is charged with murder for killing Jennifer under a statute that defines murder as "knowingly killing another human being." Heather worked as a pharmacist and gave Jennifer the wrong medication when she came to the pharmacy to have a prescription filled. The incorrect prescription was fatal to Jennifer because of a cardiac condition that she suffered from. Should Heather be convicted of murder under the statute? C (A) Yes, but only if Heather gave Jennifer the wrong medication with the purpose of killing her (B) Yes, if Heather consciously disregarded a substantial and unjustified risk of death (C) Yes, if Heather was aware that Jennifer's death was practically or virtually certain to occur (D) Yes, because medications are regulated by strict liability statutes

C Rationale: This question emphasizes the proper definition for the required mental state of knowledge. Choice A is incorrect because acting with purpose is a higher mental state than knowledge and the statute in question only requires knowledge; so it is not necessary for Heather to have the purpose of killing Jennifer to be convicted of this offense. Choice B is incorrect because conscious disregard of a substantial and unjustified risk is the definition of recklessness, not knowledge. Choice D is incorrect because strict liability has nothing to do with this situation. Although it is true that medications are often regulated by strict liability statutes, the statutory definition in this case makes clear that knowledge is the required mental state. Choice C is therefore the correct answer. To be convicted of the offense, Heather must have been aware that Jennifer's death was practically virtually certain to occur in order for her to satisfy the knowledge requirement. Recommended Reading: Pages 231-235 of Kaplan, Criminal Law: Cases and Materials, 8th Edition

Xavier hires Richard to do a job for him. Richard knows that, in the past, Xavier has been involved in the illegal drug business. Indeed, Richard knows that Xavier has been convicted for dealing narcotics on multiple occasions. Xavier tells Richard that the job involves driving a car across the country and delivering the car to another individual, who will take receipt of the car and give Richard $200,000 in cash. The car is a 1995 Chevy Malibu in poor condition. The plan is for Richard to then take an airplane back home and give the $200,000 in cash to Xavier. For his labor, Richard is scheduled to receive $15,000 in cash from Xavier. Xavier tells Richard not to let anyone look in the trunk or the merchandise in it. Xavier also tells Richard: "You might prefer not to open the trunk either if you don't want to know what's in it." The police stop the car and find drugs in it. Richard is arrested and charged with drug possession, which in this jurisdiction requires that the defendant "knowingly" possess an illegal substance. Richard claims at trial that he never knew that the trunk contained drugs and therefore he shouldn't be convicted. Is Richard's argument valid? C (A) Yes, because he did not have direct knowledge that drugs were in the car (B) No, because he knew that Xavier had been previously convicted of drug offenses (C) No, because Richard deliberately and willfully avoided obtaining the relevant information (D) Yes, because the mental state of knowledge requires practical or virtual certainty

C Rationale: This question focuses on the doctrine of willful blindness or what is sometimes also referred to as the "ostrich instruction" that judges sometimes give to jurors. In this case, Richard never looked in the trunk and so never acquired the knowledge that would be the basis for his conviction. However, the willful blindness doctrine applies in this scenario and makes a conviction possible under the mental state of knowledge because Richard suspected that there were drugs in the car and didn't open the trunk so that he wouldn't acquire the incriminating information. Consequently, Choice A is the wrong answer because although Richard did not have direct knowledge, he still can be convicted under the willful blindness doctrine. Choice B is not the best answer. Although it may be somewhat relevant that Xavier had prior drug convictions, the more relevant fact is Richard's suspicion that there were drugs in the trunk in this case, as opposed to previous encounters. Choice C is the correct answer because Richard was willfully blind and therefore under the doctrine can still be convicted of a crime of knowledge. Choice D is incorrect. Although it correctly states the standard for the mental state of knowledge (practical or virtual certainty), Choice D fails to mention the willful blindness doctrine and its application to the fact pattern. Recommended Reading: Pages 235-237 of Kaplan, Criminal Law: Cases and Materials, 8th Edition

Frank is the owner of a farm. He has a barn where he houses pigs and other farm animals, as well as farm equipment. Unfortunately, neighborhood teenagers frequently break into the barn to steal equipment or to have parties there while Frank is away from the farm. Fed up with his inability to prevent this illegal theft and trespassing, Frank builds a spring gun and attaches it to the front door. The machine is linked to the front door of the barn, so that if someone opens the door, a mechanical lever will pull the trigger of a stationary rifle that is mounted on wood planks in the center of the barn. A small sign is placed to the right of the front door of the barn that reads, "Trespassers will be shot." Frank only activates the machine when he is away from the barn. One day, an unarmed local teenager tries to break into the barn and is shot by the spring gun and dies. Which of the following statements is most true? (A) Frank is not justified because the warning sign was not placed in a conspicuous location and the teenager did not see it. (B) If Frank had been in the barn, he would have been justified in using self-defense so he was justified in using an automated machine to deploy that force. (C) Frank is not justified because the barn was an attractive nuisance. (D) Frank is not justified because he was not present in the barn.

D Rationale: Frank's use of the spring gun is problematic because he was not present. Consequently, he was not using automated force to protect himself or anyone else. Rather, he was using automated force to protect his property. Even the Castle Doctrine or Stand Your Ground laws would not apply here because those laws assume that the occupant is present at the time of the home invasion. The location—or even the existence—of the warning sign is not relevant. So Choice A is incorrect. Choice B is not the best answer because Frank's absence from the barn is critical to whether deadly force is permitted or not. Choice C is not correct because the tort doctrine of attractive nuisance has nothing to do with the law of self-defense or defense of property. Choice D is therefore the best answer. Recommended Reading: Pages 601-605 of Kaplan, Criminal Law: Cases and Materials, 8th Edition

Harry is mentally ill. One day he is sitting in the park on a bench. He sees a man walk by and feels compelled to assault him. Harry is charged with assault, but his lawyer wants the jury to find him not guilty by reason of insanity. Both prosecution and defense psychiatrists testify that Harry knew that hitting the man was wrong but suffered from a mental disease that produced an irresistible impulse to commit the assault. Which of the following statements is most correct? (A) Harry will be found not guilty if the jurisdiction applies the M'Naghten test. (B) Harry will be found guilty if the jurisdiction applies the M'Naghten test. (C) Harry will be found not guilty if the jurisdiction applies the irresistible impulse test. (D) Both B and C.

D Rationale: In order to be found not guilty by reason of insanity, it is not enough for Harry to be suffering from a mental illness. In order to meet the standard, he needs to satisfy the standard applicable in that jurisdiction. Under the M'Naghten test, the actor is not responsible if he suffered from a mental disease or defect that prevented him from understanding the nature of his act or from appreciating the wrongfulness of the action. In this case, the psychiatric testimony precludes the successful application of this defense. So Choice B is nominally correct and Choice A is incorrect. However, Choice C is also correct because Harry meets the requirement for the irresistible impulse test because the psychiatrist testifies that he was unable to control his behavior. Therefore the best answer is Choice D, which states that both B and C are correct answers.

4 Mary is 30 years old and is at home with her two-year-old daughter Sasha. Mary's new boyfriend, George, is also at the house visiting for the day. George is not the father of Sasha and does not live in the house with them. Mary is a drug addict who frequently harms Sasha by physically assaulting her. One day, Mary is particularly enraged and hits Sasha so hard that the girl dies. George does nothing to stop the assault and does not call 911 when he realizes that the girl is injured. Mary is charged with murder. Can George be charged as well? D (A) Yes, because he was in loco parentis (B) Yes, because his failure to call 911 was one cause of Sasha's death (C) No, because omissions are never punishable (D) No, because he had no legal duty to assist the girl

D Rationale: In the United States, the general rule is that omissions are not punishable unless the defendant had some duty to act. The duty to act can stem from a special relationship (parent to child, lifeguard to swimmer, doctor to patient) or by statute. In the absence of such a duty, the omission is not punishable. In this case, the boyfriend is not the child's father and therefore is unlikely to be subject to a legal duty to assist the child. (If George were a step-parent, the situation might be different.) Consequently, Choice A is incorrect because Mary was the child's parent, not George. Choice B is incorrect because the fact that George's failure to call 911 is a cause of the girl's death is irrelevant since George had no duty towards the child. Choice C is also incorrect because it is false that omissions are never punishable; in some cases they are. Choice D is the correct answer because George had no legal duty to act and therefore his omission cannot be the foundation for a criminal prosecution. Recommended Reading: Pages 124-129 of Kaplan, Criminal Law: Cases and Materials, 8th Edition

Johnson is on the subway when two young men approach him. Johnson believes that the two young men are about to violently attack him. Johnson pulls out a gun and kills both of them. His lawyer argues that he acted under a reasonable belief that he faced an imminent threat of serious bodily injury. Which of the following statements is most correct about how the jury will apply the reasonable belief standard? (A) The jury will have to determine whether Johnson's belief was objectively reasonable or not. (B) The jury will have to determine whether Johnson sincerely believed that his actions were reasonable. (C) The jury will apply a reasonable-belief standard but in doing so will consider the physical, mental, and psychological characteristics of Johnson (D) All of the above, depending on the court and the jurisdiction.

D Rationale: Jurisdictions and courts vary in how they understand and apply reasonable belief standards. Choice A is plausible because most jurisdictions understand the concept objectively, as in what a reasonable person in that situation would do. Choice B is correct because all jurisdictions require, at a minimum, that the defendant believed his defensive action was reasonable. Choice C is plausible because some courts use a reasonableness standard that requires the jury to consider many of the personal characteristics of the defendant. Consequently, the best answer is Choice D. Recommended Reading:

Victor gets incredibly drunk at a bar. After leaving the bar, Victor stumbles down the street and stops in front of a house. Victor breaks the front window of the house and crawls into the living room. Once inside, Victor stumbles around a bit before being tackled by the homeowner. Shortly thereafter, the police arrive and charge Victor with burglary. At trial, Victor claims voluntary intoxication as a defense. Which of the following statements is most true? C (A) Victor can assert the defense of voluntary intoxication, because he was charged with a specific intent crime. (B) Victor's defense will succeed if the jury is convinced that his intoxication prevented him from forming the intent to commit a felony inside the house. (C) Neither A nor B. (D) A or B, depending on the jurisdiction.

D Rationale: Many jurisdictions follow the traditional scheme of allowing voluntary intoxication as a defense only in cases where the actor is charged with a specific intent crime. However, some jurisdictions follow the more modern scheme, also codified in the MPC, which only allows voluntary intoxication as a defense if the intoxication negates the mens rea for the offense. In this case, there is a sliver of possibility that the defense might be accepted if the jury concludes that Victor was so drunk that he could not form the intent to commit a felony inside the house. In that case, the defense might be successful. Therefore, Choice D is the best answer because it would depend on the jurisdiction. Neither Choice A nor Choice B is the best answer because both are plausible answers, depending on the jurisdiction. Choice C is also wrong for this reason. Recommended Reading: Pages 280-285 of Kaplan, Criminal Law: Cases and Materials, 8th Edition

Two sisters are deeply committed to the environmental movement. Unhappy that the federal Environmental Protection Agency (EPA) is refusing to take aggressive measures to combat global climate change, the sisters decide to stage a protest at the offices of the Internal Revenue Service. The sisters break into the offices and intentionally flood the facility with water, symbolically replicating the situation that will occur when climate change causes ocean levels to rise and flood coastal cities. At trial, the sisters argue that their trespassing was justified by necessity because millions of people will be harmed, even killed, by the consequences of global climate change. Will a court allow the sisters to argue necessity? (A) Yes, because the threatened harm is severe. (B) Yes, because the sisters were engaged in direct civil disobedience. (C) No, because the threatened harm is not imminent. (D) No, because the case involves indirect civil disobedience.

D Rationale: The correct answer is Choice D. Courts are generally unwilling to entertain the necessity defense in cases of indirect civil disobedience, i.e., where the defendant violates a law other than the one they are protesting. (Courts are more willing to entertain a necessity defense in cases of direct civil disobedience, i.e., where the defendant breaks the very law that they are protesting). This case involves indirect civil disobedience because the sisters are not protesting the law against trespassing. Choices A and B are incorrect because both incorrectly suggest that the necessity defense could be successful in this case. Choice C is incorrect because necessity cases always involve harms that will occur in the future.

The City of Caroline has a problem with illegal narcotics. In the last two years, the police have responded to 300 heroin overdoses, 50 of which were fatal. Arrests for possessing and selling illegal narcotics have skyrocketed. Many of the arrests involve individuals who have prior drug convictions. The City of Caroline has decided to create a special drug court with jurisdiction over all narcotics offenses. The judges in the court use jail as a last resort and prefer to sentence the defendants to drug rehabilitation centers so that they can end their addictions, followed by community service. How can we describe the program? D (A) It treats addiction as a disease requiring treatment. (B) It focuses on rehabilitation rather than punishing defendants. (C) It is a creative solution to the problem of illegal drugs. (D) All of the above.

D Rationale: The following question focuses on criminal justice policy. Although the criminal law is usually focused on criminal punishment, it need not be. Alternative mechanisms are possible and are sometimes deployed to the great benefit of the local community. The problem of drug addiction is no exception. The justifications listed in Choices A, B, and C are all legitimate expressions of criminal justice policy. Consequently, Choice D is the best answer. Recommended Reading: Pages 47-50 of Kaplan, Criminal Law: Cases and Materials, 8th Edition

Karl is working in his lab when he sees his boss, Nathan, walk in and lock the door, holding a bomb in his hands. Karl knows that Nathan is an expert bomb-maker, and he has just decoded a computer file in which Nathan described the bomb, as follows: it is programmed to explode 24 hours after Nathan activates it, and there is no way to diffuse it once it is activated. Nathan is not holding any other weapon or making any other threatening actions other than holding the bomb. However, only Nathan has a key to the door, and Karl has no means of communicating with anyone outside. In order to prevent Nathan from activating the bomb, Karl shoots and kills Nathan. Is Karl eligible for the justification of self-defense? (A) No, because the threat of death was not imminent (B) Yes, if the jury determines that the shooting was immediately necessary to stop the threat (C) Neither A nor C (D) A or B, depending on the jurisdiction

D Rationale: The temporal requirement for self-defense depends on the jurisdiction. Most jurisdictions require that the actor be facing an imminent threat. However, the Model Penal Code requires that the action be immediately necessary to resolve the threat, which focuses more on whether the defensive force was required at that moment in time to alleviate the threat. Consequently, Choice A and Choice B are both incomplete answers. Choice D is the best answer because the standard depends on whether the jurisdiction applies the Model Penal Code formulation or not. In this case, it is at least possible for a jury to determine that, although the threat was not imminent, Karl's action was immediately necessary to stop the threat—Karl could reasonably believe that shooting Nathan at that moment was his last chance to stop the threat. Recommended Reading: Pages 557-561 of Kaplan, Criminal Law: Cases and Materials, 8th Edition

Jackson is walking down a dark alley. In the distance, he sees a teenager attacking a homeless man. The attack looks vicious and it appears that the homeless man is on the verge of being substantially injured or even killed. Jackson screams at the teenager to stop, but the teenager ignores the verbal injunction. Fearing that the homeless man is facing imminent death or serious bodily injured, Jackson pulls out a gun and shoots the teenager. Which of the following statements is most true? (A) Jackson was not justified because he was not personally facing an imminent threat from the teenager. (B) The homeless man had a right of response to defend himself but Jackson was not entitled to exercise that defensive right for him. (C) Both A and B. (D) The legal conditions for Jackson's justification are the same as self-defense except that the target of the threat was the homeless man, not Jackson.

D Rationale: This fact pattern presents a "defense of others" situation. Jackson is entitled to assert a justification for defending someone else from an imminent threat. In most states, the legal criteria for defense of others is symmetrical with the criteria for self-defense, in the sense that an actor is allowed to exercise defense of others on behalf of any individual who has the right to exercise self-defense on their own behalf. Consequently, Choices A and B are both incorrect because both imply that defense of others is not a viable justification. Choice C is incorrect for the same reason. So Choice D is the best answer because Jackson will be entitled to exercise force on the man's behalf, as long as he had a reasonable belief in an imminent threat and the force was necessary to repel the assault. Recommended Reading: Page 560 of Kaplan, Criminal Law: Cases and Materials, 8th Edition, and Model Penal Code Sec. 3.05

Jeremiah, Markus, and Billy are three teenagers at home on a Sunday afternoon. Jeremiah has a gun. He hands it to Markus and tells Markus that the gun is unloaded. Markus and Billy start playing with the gun. At some point, Markus points the gun at Billy and pulls the trigger. The gun discharges and a bullet strikes Billy in the head, killing him instantly. Markus is charged with involuntary manslaughter, which in this jurisdiction requires that the defendant recklessly caused the death of another human being. The prosecutor's theory of the case is that Markus knew he should have checked the gun to ensure that it wasn't loaded, which he didn't do. Markus wants to argue that he cannot be convicted because he was laboring under a mistake of fact. Is Markus right about mistake of fact? Does his mistake automatically preclude his conviction for manslaughter? This jurisdiction follows the Model Penal Code provision on mistakes. D (A) Yes, because he was unaware that the gun was loaded (B) Yes, because Jeremiah is more responsible than Markus for the death of Billy (C) No, because a jury could find that his mistake constituted his recklessness, rather than negating it (D) No, because mistake of fact defenses aren't available in manslaughter cases

D Rationale: This is a complex case. Under the MPC, mistakes of fact are evaluated by first determining the required mental state for the offense. In this case, manslaughter requires that the defendant consciously disregard a substantial and unjustified risk. So the question is whether the defendant's mistake negates that mental element. In this case, the defendant's mistake does not negate it. Instead, the defendant's mistake is actually evidence of his recklessness because a careful individual (according to the prosecutor) would have checked the gun to ensure that it was unloaded before pointing the weapon and firing it. Consequently, Choice C is the correct answer. Choice A is incorrect because even though Markus was unaware that the gun was loaded, that isn't the relevant question here. It would be relevant if Markus had been charged with intentional murder, but he is charged with manslaughter in this case. Choice B is incorrect because the culpability of the other parties has no bearing on Markus's mistake of fact argument. Choice D is also incorrect because its statement is far too categorical—there could be situations where a mistake of fact is relevant to a manslaughter case. Recommended Reading: Pages 202-205 of Kaplan, Criminal Law: Cases and Materials, 8th Edition; and MPC Sec. 2.02(2)(c)

4 Jason is prosecuted for the offense of unlawfully receiving state welfare benefits. At trial, the judge tells the jury that the crime requires no mental state because the statute does not list one. Jason is prosecuted and sentenced to 10 years in prison. Jason appeals and argues that the judge was wrong to instruct the jury that the crime does not require a mental state. In making this determination, which factor will not be relevant to the court's consideration of the issue? (A) The punishment for the crime is 10 years in prison. (B) Whether the statute is a codification of a common law crime that was strict liability. (C) Whether the offense comes with substantial social opprobrium attached to it. (D) Whether Jason understood that the crime was strict liability when he engaged in the action.

D Rationale: This question focuses on the judicial methodology for how courts determine whether a criminal statute is strict liability or not. Choice A is not the best answer because courts often look to the length of the prison sentence attached to the offense to determine if it is strict liability. If the statute allows for the possibility of severe punishment, the court is less likely to view it as strict liability. Choice B is also not the best answer because courts frequently look to the common law antecedent of the statute—if there is one—to determine the appropriate mental state (if any) under the statute. Choice C is also not the best answer because whether the crime comes with social opprobrium is highly relevant to determining whether the crime is strict liability or not. Finally, Choice D is the best answer because it is not relevant that Jason understood the crime as a strict liability one or not. In short, Choices A, B, and C all involve highly relevant factors, so Choice D is the only viable answer. Recommended Reading: Pages 210-211 of Kaplan, Criminal Law: Cases and Materials, 8th Edition

Billy is driving down the street in his sedan. He is speeding, crosses the median into oncoming traffic, and hits a car traveling in the opposite direction. A passenger in that car is severely injured. Billy is not injured. Billy does not assist the passenger and does not call 911. There is a pedestrian walking nearby who sees the accident but refuses to render assistance or call 911. The prosecutor wants to charge someone for his or her failure to act. The state legislature recently passed a new statute that requires anyone witnessing an accident to call 911 if he or she can safely do so without personal risk. Who can the prosecutor charge? D (A) Neither, because the statute is unconstitutional (B) Billy only, because he caused the accident (C) The pedestrian only, because the statute applies to him (D) Both, because Billy had a duty because he caused the accident and the pedestrian had a statutory duty

D Rationale: This question requires consideration of two independent bases for omission liability. One is the duty to act in questions where the defendant is responsible for the harm in the first instance. The second is duties that are imposed by statute. In this fact pattern, Billy has a duty under the first and the pedestrian has a duty under the second. Consequently, Choice A is incorrect because there is nothing in the U.S. Constitution that prevents a state from imposing a statutory duty to act. Choice B is also not the best answer because it is only half correct; the pedestrian has a duty as well. Choice C is incorrect for similar reasons; it fails to note that Billy is responsible for his omission as well. Consequently, Choice D is the best answer because it explicitly articulates the two independent bases for each actor's duty to act. Recommended Reading: Pages 124-129 of Kaplan, Criminal Law: Cases and Materials, 8th Edition

Manuel is the manager of a food processing plant. The plant manufactures, among other things, peanut butter. Manuel learns that some of the peanut butter produced in the facility may have been contaminated with dangerous bacteria. After the peanut butter is shipped to stores and sold, 12 people become seriously ill and one of them dies from eating the tainted peanut butter. The local prosecutor has launched a criminal case against Manuel in a jurisdiction that follows the Model Penal Code. Can Manuel be convicted of a crime of recklessness? D (A) Yes, because he should have been aware of the substantial and unjustified risk associated with releasing the peanut butter (B) No, because it was not virtually or practically certain that someone would be harmed by the peanut butter (C) No, because Manuel did not intend for anyone to get hurt by the peanut butter (D) Yes, but only if he consciously disregarded a substantial and unjustified risk

D Rationale: This question requires the reader to understand and apply the MPC definition for recklessness, which would govern prosecutions for involuntary manslaughter in many jurisdictions. Choice A is incorrect because if Manuel "should have been aware" of the risk, then the most he can be convicted of is a crime of negligence, not recklessness, under the MPC scheme. Choice B is incorrect because being aware of a virtual or practical certainty of an event occurring is the MPC standard for acting with knowledge, which is a higher mental state. Choice C is not correct because recklessness does not require that the defendant intend the result; it is enough that the defendant is aware of the possibility that the harm might occur yet consciously disregards the risk and engages in the action anyway. Choice D is therefore correct. If Manuel consciously disregarded a substantial and unjustified risk associated with the tainted peanut butter, he meets the standard for acting recklessly under the Model Penal Code. Recommended Reading: Pages 231-239 of Kaplan, Criminal Law: Cases and Materials, 8th Edition

Barbara works at a jewelry store. One day, an escaped convict rushes into the store and pulls a gun on Barbara. The convict threatens to kill Barbara unless Barbara steals some jewelry for him. The convict then threatens to kill Barbara unless Barbara kills a co-worker. Fearing death, Barbara kills the co-worker. Barbara is charged with theft and murder. Which of the following statements is most accurate, assuming that the jurisdiction does not apply the Model Penal Code? (A) Barbara is guilty of neither theft nor murder. (B) Barbara is guilty of theft but not murder. (C) Barbara is guilty of murder but not theft. (D) Barbara is guilty of both murder and theft.

Rationale: Barbara's situation fits the paradigm for duress: a third party is forcing her to violate the law by threatening death or serious bodily injury. The standard rule for both necessity and duress is that neither defense is available for the crime of murder—a rule stemming from the famous Dudley and Stephens case but also applicable to the defense of duress. However, the Dudley and Stephens restriction on applying the defense to crimes of murder would not prevent Barbara from asserting the defense for the crime of theft. Therefore, Choice C is the best answer. Choice A is incorrect because Barbara has no defense to murder. Choices B and D are incorrect because Barbara has no defense to theft. It is important to note that the prohibition on using the duress defense in murder cases does not appear in the Model Penal Code (2.09).

The defendant is charged with assaulting a person on the street. The defendant argues at trial that he only committed the crime because a third person—the boss of a criminal gang—threatened to kill the defendant unless he committed the assault for the gang. The defendant says that he reluctantly complied because he feared for his life. In a jurisdiction that follows the traditional common-law rule, does the defendant bear the burden of establishing the defense or does the prosecution bear the burden of disproving it? (A) The prosecution must disprove the defense beyond a reasonable doubt. (B) The prosecution must disprove the defense by a preponderance of the evidence. (C) The defense must prove the defense by a preponderance of the evidence. (D) The defense must prove the defense beyond a reasonable doubt.

Rationale: Choice C is the correct answer. The common-law rule is that the defendant bears the burden of establishing the defense of duress by a preponderance of the evidence. Choices A and B are incorrect because the prosecution does not bear the burden. Choice D is incorrect because it misstates the standard of proof for the defense. Recommended Reading:

A prisoner is housed in a maximum-security prison. The prisoner is subjected to a series of increasingly violent attacks from his cellmate. The prisoner reports these attacks to prison officials, but their response is inadequate. The prisoner requests that he be segregated from the violent cellmate, but the prison officials refuse the request. The attacks continue until the inmate hatches an elaborate plan to escape from the prison in order to get away from the cellmate. The inmate succeeds in the escape, changes his appearance, and stays in a motel under an assumed name for six months. After six months, the police arrest him and charge him with escaping from custody. At trial, he asserts a necessity defense. Does the necessity defense apply in this case? (A) Yes, because it was necessary to escape the threat. (B) Yes, because prison officials refused to protect him. (C) No, because the threat evaporated once he had escaped the prison. (D) No, because the necessity defense never applies in the prison escape context.

Rationale: Choice C is the correct answer. The necessity defense offers a justification when the defendant needs to break the law in order to avoid a threatening situation. In this case, the threat was posed by the cellmate, but once the prisoner had escaped the prison, he was no longer being threatened by the cellmate. At that point, his continued escape from custody was unjustified. Choices A and B are both incorrect because they falsely suggest that the prisoner would be justified by necessity for the entire period of his escape. Choice D is incorrect because it falsely suggests that no prisoner could ever be justified by necessity for escaping a prison. Recommended Reading: Pages 616-621 of Kaplan, Criminal Law: Cases and Materials, 8th Edition

Thomas is a farmer who is driving his tractor across a field. Thomas gets into an accident when he collides with a neighboring farmer's tractor on a shared dirt road. The collision results in Thomas's tractor getting overturned in a muddy ditch with the arm of the other farmer trapped under Thomas's tractor. Thomas tries to move the tractor out of the muddy ditch despite the fact that he knows that doing so will result in catastrophic damage to the other farmer's arm, requiring its amputation. Thomas is charged with assault. At trial, Thomas argues that his actions were justified by necessity because they were required in order to protect his property (his tractor) from permanent damage. Which of the following statements is most correct? (A) Necessity is never a justification for assault. (B) Necessity is available as a defense, because the harm avoided was greater than the harm imposed. (C) Necessity is not available as a defense, because the harm imposed was greater than the harm avoided. (D) Necessity does not require that the actor choose the lesser of two evils.

Rationale: Necessity, also known as the "choice of evils" defense, requires that the actor select the lesser of two evils. In this case, although Thomas was motivated by necessity, his actions were disproportionate. He protected his own personal property by imposing harm on someone's bodily integrity—essentially imposed an assault in order to preserve property. This was not the lesser of two evils. Consequently, Choice C is the correct answer. Choice A is incorrect because necessity could, in theory, be a justification for assault in the right circumstances. Choice B is incorrect because in this case the harm avoided was not greater than the harm imposed. Choice D is incorrect because the necessity defense does require that the actor choose the lesser of two evils-hence the alternate name for the defense.

Donald receives a phone call from an anonymous caller who refuses to identify himself. Donald is directed by the caller to seriously assault his neighbor by breaking the neighbor's arm with a baseball bat. The caller tells Donald that if Donald fails to comply with the demand, the caller will track down and find Donald's best friend and "punch him real hard in the gut." Fearful of his best friend being punched, Donald succumbs to the threat and assaults his neighbor with a bat. Donald is charged with assault but wishes to assert the duress defense. Which statement is most correct? (A) The duress defense can never apply in cases of serious assault. (B) Under the Model Penal Code and in some jurisdictions, Donald will not be excused by duress if the jury concludes that a person of reasonable firmness in this situation would have been able to resist the threat. (C) Donald is not excused by duress because the assault on his neighbor was not the lesser of two evils. (D) None of the above.

Rationale: Only some threats will generate a duress defense. Under the Model Penal Code, if a person of reasonable firmness would be capable of resisting the threat, then the excuse does not apply. In this case, it seems plausible that a jury might conclude that a person of reasonable firmness would be able to resist the threat of a friend simply being punched (as opposed to being injured more severely). Consequently, Choice B is the best answer. Choice A is incorrect because there is no bright line rule regarding duress and assault. Choice C is also not the best answer because, although the assault on the neighbor was not the lesser of two evils, duress generally does not require a balancing of e

A man is a low-level worker in an organized crime gang. The criminal organization is focused on running an illegal betting operation. The boss of the criminal organization is unhappy about a rival betting business. The boss tells the low-level worker to murder the rival. When the low-level worker says no, the boss threatens to kill the low-level worker unless he carries out the crime. Fearful of these threatened consequences, the low-level worker murders the rival. In which jurisdictions can the low-level worker assert a defense of duress? (A) In jurisdictions that follow the Model Penal Code. (B) In jurisdictions that follow the common-law rule for duress. (C) In jurisdictions that follow the MPC and jurisdictions that follow the common-law rule. (D) In no jurisdiction.

Rationale: The correct answer is Choice A. The traditional common-law rule is that duress is not available as a defense to murder. Consequently, Choices B and C are both incorrect because they state that the defense is available in common-law jurisdictions. Choice A is the correct choice because the Model Penal Code provision on duress contains no exclusion with regard to the offense of murder. Choice D is incorrect because if the defense of duress would be applicable in a jurisdiction that follows the Model Penal Code, it cannot be correct to state that no jurisdiction would allow the defense.

Daniel is on trial for murder. At the conclusion of the trial, the judge instructs the jury prior to deliberations. The judge explains to the jury that in order to find the defendant guilty, the prosecutor must establish, beyond a reasonable doubt, each material element of the offense of murder. The judge then offers the jury the following definition of reasonable doubt: "You should be convinced that it is at least 55 percent certain that each material element has been proven." This jury instruction is (A) Clumsy but probably not likely to be overturned on appeal. (B) The correct definition of reasonable doubt. (C) Unconstitutional, because judges should not quantify the reasonable doubt standard in a way that waters it down. (D) Constitutional because judges have unfettered discretion to define terms during jury instruction.

(C) Unconstitutional, because judges should not quantify the reasonable doubt standard in a way that waters it down. Although the reasonable doubt standard is often mysterious, judges are frequently overturned on appeal if they offer a definition that effectively changes the standard. One example of this problem is quantifying the standard by attaching fixed numbers to it. Consequently, Choice C is the best answer because the instruction is probably unconstitutional. Choice A is not the best answer because the instruction is likely to be overturned. Choice B is also incorrect because "55 percent" is not a standard or correct definition of reasonable doubt. Choice D is patently false because judges do not have unfettered discretion to define legal terms during their charge to the jury. In fact, appeals courts frequently overturn trial judges when the trial judge misstates the law during the jury charge.

2 Dennis is on trial for murder. In her closing statement, the prosecutor says that the jury should convict the defendant if his lawyer has not managed to disprove the prosecutor's case. The judge, in instructing the jury, says something similar: The jury should convict if the defense has not established by a preponderance of the evidence that the defendant could not have committed the crime. The jury convicts Dennis of murder. On appeal, Dennis argues that his conviction should be overturned because it violated his right to a jury trial. Can Dennis prevail on this argument? (A) Yes, because the judge's instruction referred to preponderance of the evidence, which is the standard for civil cases, not criminal cases (B) No, because the defendant always has the burden of proof in criminal cases (C) Yes, because the jury should have been told that the prosecution has the ultimate burden to demonstrate, beyond a reasonable doubt, each material element of the crime charged (D) No, because the burden of proof is neither quantifiable nor justiciable on appeal as a matter of law

(C) Yes, because the jury should have been told that the prosecution has the ultimate burden to demonstrate, beyond a reasonable doubt, each material element of the crime charged Rationale: The answer depends on properly understanding the burden of proof in criminal cases. The ultimate burden to persuade the jury rests with the prosecution. So Choice A is not the best answer. Although it is true that the judge should not have referred to "preponderance of the evidence," which is the standard in civil cases, the real reason the judge's instruction was problematic was that he improperly assigned the ultimate burden to the defense when in fact it resides with the prosecution. Choice B is incorrect because the ultimate burden rests with the prosecution. Choice C is clearly correct: the jury should have been told that the prosecution must establish, beyond a reasonable doubt, each material element of the crime. Choice D is inapposite. While it is true that in some sense standards such as "beyond a reasonable doubt" defy easy quantification in numerical terms, it is certainly not the case that complaints about how the jury is instructed are non-justiciable on appeal. Recommended Reading: Pages 16-17 and 359-62 of Kaplan, Criminal Law: Cases and Materials, 8th Edition

Jeremiah is the owner of a delicatessen. A young man walks in to the shop and Jeremiah, believing that the young man is committing a robbery, shoots and kills him. The local prosecutor conducts an investigation and determines that Jeremiah's use of force was inappropriate. Jeremiah is arrested and charged with murder. Which statement below is most accurate? (A) In most states, Jeremiah has the initial burden of production as to self defense, but the prosecution must then disprove the defense beyond a reasonable doubt. (B) In a few states the defendant must prove self-defense by a preponderance of the evidence. (C) There is no constitutional prohibition preventing a state legislature from allocating the burden of proof, on self defense, to the defendant. (D) All of the above.

(D) All of the above. Rationale: In the vast majority of US states, the defendant bears the initial burden of production, to establish some facts tending to show that the defendant acted in self-defense. Once that burden is met, the burden shifts to the prosecution to disprove the defense beyond a reasonable doubt. Consequently, Choice A is correct. However, Ohio does not follow this rule, and in Martin v. Ohio the Supreme Court upheld Ohio's scheme, so Choice B and Choice C are also both correct. Consequently, the best answer is Choice D, all of the above. Recommended Reading: Page 18 of Kaplan, Criminal Law: Cases and Materials, 8th Edition

The state legislature has just passed a new statute that changes its law of theft. Prior to the passing of the new statute, the definition of theft was elucidated in judicial decisions arising from prosecutions of defendants for the crime of theft. However, several legislators disliked the definition in those cases and organized an effort to pass the new statute. In passing the new statute, the state legislature borrowed the definition of theft found in the Model Penal Code, though the legislature tinkered with the definition slightly and specifically excluded parts of the Model Penal Code definition. In a future case, if there is an ambiguity in the statute, will a court consult the Model Penal Code or the older common law definition or both? (A) The Model Penal Code, because it is newer (B) The common law definition, because it is older (C) None of them, because the new statute differs from both the common law definition and the Model Penal Code definition (D) Both of them

(D) Both of them Rationale: This question focuses on judicial methodology rather than on the substance of the doctrine. In this situation, which is not uncommon, the state changed its law and abandoned the older common law definition. In making the change, the state looked to the Model Penal Code but did not adopt the MPC definition in its entirety. Choice A is not the best answer because it is only partially correct. True, the court will look to the Model Penal Code, but this is not the only source it will consult. Likewise, Choice B is not the best answer because it falsely suggests that the Model Penal Code is irrelevant to the court's investigation, which cannot be the case because the new law is mostly based on the Model Penal Code formulation. Choice C is incorrect because, although the new definition differs in some respects from both the Model Penal Code definition and the older common law definition, the court will want to look to these sources to understand why the state legislature departed from them. Consequently, Choice D is correct because the court will likely consult both of these sources to determine why the state legislature abandoned the common law definition but also why it failed to adopt the Model Penal Code provision in its entirety. By looking at the alternatives that the state legislature rejected, the court can gain insight into the meaning and purpose of the provision that the legislature drafted. Recommended Reading: Pages 9-10, 14-15, and 208-210 of Kaplan, Criminal Law: Cases and Materials, 8th Edition


Conjuntos de estudio relacionados

Elbow and Distal Humerus Chapter 5

View Set

Chapter 5. Freedom of Expression

View Set

A Good Man is Hard to Find - Flannery O'Connor

View Set

Chapter 5: Biomes and Biodiversity

View Set